Practice Questions Flashcards

1
Q

Which of the following best describes asthma?
A. intermittent airway inflammation with occasional bronchospasm
B. a disease of bronchospasm that leads to airway inflammation
C. chronic airway inflammation with superimposed bronchospasm
D. relatively fixed airway constriction

A

C. chronic airway inflammation with superimposed bronchospasm

How well did you know this?
1
Not at all
2
3
4
5
Perfectly
2
Q
The patient you are evaluating is having a severe asthma flare. You have assessed that his condition is appropriate for office treatment. You expect to find the following on physical examination:
A. tripod posture
B. inspiratory crackles
C. increased vocal fremitus
D. hyperresonance on thoracic percussion
A

D. hyperresonance on thoracic percussion

How well did you know this?
1
Not at all
2
3
4
5
Perfectly
3
Q

A 44-year-old man has a long-standing history of moderate persistent asthma that is normally well controlled by fluticasone with salmeterol (Advair) via metered-dose inhaler, one puff twice a day, and the use of albuterol 1 to 2 times a week as needed for wheezing. Three days ago, he developed a sore throat, clear nasal discharge, body aches, and a dry cough. In the past 24 hours, he has had intermittent wheezing that necessitated the use of albuterol, two puffs every 3 hours, which produced partial relief. Your next most appropriate action is to obtain a:
A. chest radiograph.
B. measurement of oxygen saturation (SaO2).
C. spirometry measurement.
D. sputum smear for white blood cells (WBCs).

A

C. spirometry measurement.

How well did you know this?
1
Not at all
2
3
4
5
Perfectly
4
Q
You examine Jane, a 24-year-old woman who has an acute asthma flare following a 3-day history of upper respiratory tract symptoms (clear nasal discharge, dry cough, no fever). She has a history of moderate persistent asthma that is in good control and an acceptable peak expiratory flow (PEF). She is using budesonide (Pulmicort) and albuterol as directed and continues to have difficulty with coughing and wheezing. At home, her PEF is 55% of personal best. In the office, her forced expiratory volume at 1 second (FEV1) is 65% of predicted. Her medication regimen should be adjusted to include:
A. theophylline.
B. salmeterol (Serevent).
C. prednisone.
D. montelukast (Singulair).
You also prescribe:
A. amoxicillin.
B. azithromycin.
C. levofloxacin.
D. no antimicrobial therapy.
A

C. prednisone.

D. no antimicrobial therapy.

How well did you know this?
1
Not at all
2
3
4
5
Perfectly
5
Q

Peak expiratory flow meters:
A. should only be used in the presence of a medical professional.
B. provide a convenient method to check lung function at home.
C. are as accurate as spirometry.
D. should not be used more than once daily.

A

B. provide a convenient method to check lung function at home.

How well did you know this?
1
Not at all
2
3
4
5
Perfectly
6
Q
Which of the following is most likely to appear on a chest radiograph of a person during an acute severe asthma attack?
A. hyperinflation
B. atelectasis
C. consolidation
D. Kerley B signs
A

A. hyperinflation

How well did you know this?
1
Not at all
2
3
4
5
Perfectly
7
Q
A 36-year-old man with asthma also needs antihypertensive therapy. Which of the following products should you avoid prescribing?
A. hydrochlorothiazide
B. propranolol
C. amlodipine
D. enalapril
A

B. propranolol

How well did you know this?
1
Not at all
2
3
4
5
Perfectly
8
Q

Which of the following is inconsistent with the presentation of asthma that is not well controlled?
A. a troublesome nocturnal cough at least 2 nights per week
B. need for albuterol to relieve shortness of breath at least twice a week
C. morning sputum production
D. two or more exacerbations/year requiring oral corticosteroids

A

C. morning sputum production

How well did you know this?
1
Not at all
2
3
4
5
Perfectly
9
Q
The cornerstone of moderate persistent asthma drug therapy is the use of:
A. oral theophylline.
B. mast cell stabilizers.
C. short-acting beta2 agonists (SABA).
D. inhaled corticosteroids.
A

D. inhaled corticosteroids.

How well did you know this?
1
Not at all
2
3
4
5
Perfectly
10
Q

Sharon is a 29-year-old woman with moderate persistent asthma. She is not using prescribed inhaled corticosteroids, but is using albuterol PRN to relieve her cough and wheeze with reported satisfactory clinical effect. Currently she uses about two albuterol metered-dose inhalers per month and is requesting a prescription refill. You consider that:
A. her asthma is well controlled and albuterol use can continue.
B. excessive albuterol use is a risk factor for asthma death.
C. her asthma is not well controlled and salmeterol (Serevent) should be added to relieve bronchospasm and reduce her albuterol use.
D. her asthma has better control with albuterol than inhaled corticosteroids.

A

B. excessive albuterol use is a risk factor for asthma death.

How well did you know this?
1
Not at all
2
3
4
5
Perfectly
11
Q

In the treatment of asthma, leukotriene receptor antagonists should be used as:
A. controllers to prevent bronchospasm.
B. controllers to inhibit inflammatory responses.
C. relievers to treat acute bronchospasm.
D. relievers to treat bronchospasm and inflammation.

A

B. controllers to inhibit inflammatory responses.

How well did you know this?
1
Not at all
2
3
4
5
Perfectly
12
Q

According to the National Asthma Education and Prevention Program Expert Panel Report-3 (NAEPP EPR-3) guidelines, which of the following is not a risk for asthma death?
A. hospitalization or an emergency department visit for asthma in the past month
B. current use of systemic corticosteroids or recent withdrawal from systemic corticosteroids
C. difficulty perceiving airflow obstruction or its severity
D. rural residence

A

D. rural residence

How well did you know this?
1
Not at all
2
3
4
5
Perfectly
13
Q
An 18-year-old high school senior presents, asking for a letter stating that he should not participate in gym class because he has asthma. The most appropriate action is to:
A. write the note because gym class participation could trigger asthma symptoms.
B. excuse him from outdoor activities only to avoid pollen exposure.
C. assess his level of asthma control and make changes in his treatment plan if needed so he can participate.
D. write a note excusing him from gym until his follow-up exam in 2 months.
A

C. assess his level of asthma control and make changes in his treatment plan if needed so he can participate.

How well did you know this?
1
Not at all
2
3
4
5
Perfectly
14
Q

You see a 34-year-old man with moderate persistent asthma who has a severe asthma flare and a regimen of oral prednisone is being considered. Which of the following is true?
A. A taper is needed for prednisone therapy lasting longer than 4 days.
B. A taper is not needed if the prednisone regimen is for 7 days or less.
C. A taper is not needed regardless of duration of prednisone therapy.
D. A taper is needed if the patient is taking concomitant inhaled corticosteroids.

A

B. A taper is not needed if the prednisone regimen is for 7 days or less.

How well did you know this?
1
Not at all
2
3
4
5
Perfectly
15
Q
After inhaled corticosteroid is initiated, improvement in control is usually seen:
A. on the first day of use.
B. within 2 to 8 days.
C. in about 3 to 4 weeks.
D. in about 1 to 2 months.
A

B. within 2 to 8 days.

How well did you know this?
1
Not at all
2
3
4
5
Perfectly
16
Q

Compared with albuterol, levalbuterol (Xopenex) has:
A. a different mechanism of action.
B. the ability potentially to provide greater bronchodilation with a lower dose.
C. an anti-inflammatory effect similar to that of an inhaled corticosteroid.
D. a contraindication to use in elderly patients.

A

B. the ability potentially to provide greater bronchodilation with a lower dose.

How well did you know this?
1
Not at all
2
3
4
5
Perfectly
17
Q

Which of the following is consistent with the NAEPP comment on the use of inhaled corticosteroids (ICS) for a child with asthma?
A. The potential but small risk of delayed growth with ICS is well balanced by their effectiveness.
B. ICS should be used only if leukotriene modifiers fail to control asthma.
C. Permanent growth stunting is consistently noted in children using ICS.
D. Leukotriene modifiers are equal in therapeutic effect to the use of a long-acting beta2-agonist.

A

A. The potential but small risk of delayed growth with ICS is well balanced by their effectiveness.

How well did you know this?
1
Not at all
2
3
4
5
Perfectly
18
Q
A potential adverse effect from ICS use is:
A. oral candidiasis.
B. tachycardia.
C. gastrointestinal upset.
D. insomnia.
A

A. oral candidiasis.

How well did you know this?
1
Not at all
2
3
4
5
Perfectly
19
Q

Clinical findings characteristic of asthma include all of the following except:
A. a recurrent spasmodic cough that is worse at night.
B. recurrent shortness of breath and chest tightness with exercise.
C. a congested cough that is worse during the day.
D. wheezing with and without associated respiratory infections.

A

C. a congested cough that is worse during the day.

How well did you know this?
1
Not at all
2
3
4
5
Perfectly
20
Q
Which of the following best describes the mechanism of action of short-acting beta2-agonists?
A. reducer of inflammation
B. inhibition of secretions
C. modification of leukotrienes
D. smooth muscle relaxation
A

D. smooth muscle relaxation

How well did you know this?
1
Not at all
2
3
4
5
Perfectly
21
Q

Regarding the use of long acting beta2-agonists (LABAs), which of the following is not true?
A. LABAs enhance the antiinflammatory action of corticosteroids.
B. Use of LABAs is associated with a small increase in risk of asthma death.
C. LABA use reduces the risk of asthma exacerbations.
D. LABAs can be used as monotherapy to relieve bronchospasms in asthma.

A

D. LABAs can be used as monotherapy to relieve bronchospasms in asthma.

How well did you know this?
1
Not at all
2
3
4
5
Perfectly
22
Q

Which of the following is the therapeutic objective of using inhaled ipratropium bromide?
A. as an antiinflammatory.
B. an increase in vagal tone in the airway
C. inhibition of muscarinic cholinergic receptors
D. an increase in salivary and mucous secretions

A

C. inhibition of muscarinic cholinergic receptors

How well did you know this?
1
Not at all
2
3
4
5
Perfectly
23
Q

Which of the following is true regarding the use of systemic corticosteroids in the treatment of asthma?
A. Frequent short bursts are preferred over daily inhaled corticosteroids.
B. The oral corticosteroid should be started at day 3-4 of the asthma flare for optimal effect.
C. The oral route is preferred over parenteral therapy.
D. The adult dose to treat an asthma flare should not exceed the equivalent of prednisone 40 mg daily.

A

C. The oral route is preferred over parenteral therapy.

How well did you know this?
1
Not at all
2
3
4
5
Perfectly
24
Q

Compared with SABAs, LABAs:
A. are recommended as a first-line therapy in mild intermittent asthma.
B. have a significantly different pharmacodynamic profile.
C. have a rapid onset of action across the drug class.
D. should be added to therapy only when ICS use does not provide adequate asthma control.

A

D. should be added to therapy only when ICS use does not provide adequate asthma control.

How well did you know this?
1
Not at all
2
3
4
5
Perfectly
25
Q

Which of the following statements is false regarding the use of omalizumab (Xolair)?
A. Its use is recommended for patients with mild persistent asthma to prevent asthma flares.
B. The medication selectively binds to IgE to reduce exacerbations.
C. Labeled indication is for patients with poorly controlled asthma with frequent exacerbations.
D. Special evaluation is required prior to its use and ongoing monitoring is needed during use.

A

A. Its use is recommended for patients with mild persistent asthma to prevent asthma flares.

How well did you know this?
1
Not at all
2
3
4
5
Perfectly
26
Q

Subcutaneous immunotherapy is recommended for use in patients:
A. with well-controlled asthma and infrequent exacerbations.
B. with allergic-based asthma.
C. with moderate persistent asthma who are intolerant of ICS.
D. with poorly-controlled asthma who fail therapy with omalizumab.

A

B. with allergic-based asthma.

How well did you know this?
1
Not at all
2
3
4
5
Perfectly
27
Q

T/F Most prescribers are well versed in the relative potency of ICS and prescribe an appropriate dose for the patient’s clinical presentation.

A

False

How well did you know this?
1
Not at all
2
3
4
5
Perfectly
28
Q

T/F Approximately 80% of the dose of an ICS is systemically absorbed.

A

False

How well did you know this?
1
Not at all
2
3
4
5
Perfectly
29
Q

T/F Leukotriene modifiers and ICS are interchangeable clinically because both groups of medications have equivalent anti-inflammatory effect.

A

False

How well did you know this?
1
Not at all
2
3
4
5
Perfectly
30
Q

T/F Little systemic absorption of mast cell stabilizers occurs with inhaled or intranasal use.

A

True

How well did you know this?
1
Not at all
2
3
4
5
Perfectly
31
Q

T/F Due to safety concerns, mast cell stabilizers are no longer available.

A

False

How well did you know this?
1
Not at all
2
3
4
5
Perfectly
32
Q

When discussing immunizations with a 67-year-old woman with chronic obstructive pulmonary disease (COPD), you advise that she:
A. receive live attenuated influenza virus vaccine.
B. avoid immunization against influenza because of the risk associated with the vaccine.
C. receive inactivated influenza virus vaccine.
D. take an antiviral for the duration of the influenza season.

A

C. receive inactivated influenza virus vaccine.

How well did you know this?
1
Not at all
2
3
4
5
Perfectly
33
Q

T/F Seasonal influenza vaccination is generally recommended for all persons over the age of 6 months.

A

True

How well did you know this?
1
Not at all
2
3
4
5
Perfectly
34
Q

T/F A 66-year-old woman is an acceptable candidate

for the high-dose inactivated influenza vaccine shot.

A

True

How well did you know this?
1
Not at all
2
3
4
5
Perfectly
35
Q

T/F Cigarette smokers should not receive the pneumococcal vaccine until 65 years of age.

A

False

How well did you know this?
1
Not at all
2
3
4
5
Perfectly
36
Q

T/F A 52-year-old immunocompetent patient with COPD who receives the pneumococcal vaccine should get revaccinated in 5 years.

A

False. He should receive another dose when he turns 65 plus additional doses later.

How well did you know this?
1
Not at all
2
3
4
5
Perfectly
37
Q
When used in treating COPD, ipratropium bromide (Atrovent) is prescribed to achieve which of the following therapeutic effects?
A. increase mucociliary clearance
B. reduce alveolar volume
C. bronchodilation
D. mucolytic action
A

C. bronchodilation

How well did you know this?
1
Not at all
2
3
4
5
Perfectly
38
Q

What is the desired therapeutic action of inhaled corticosteroids when used to treat COPD?
A. reversal of fixed airway obstruction
B. improvement of central respiratory drive
C. reduction of airway inflammation
D. mucolytic activity

A

C. reduction of airway inflammation

How well did you know this?
1
Not at all
2
3
4
5
Perfectly
39
Q

Which is most consistent with the diagnosis of COPD?
A. FEV1/FVC ratio equal to or less than 0.70 after properly timed SABA use
B. dyspnea on exhalation
C. elevated diaphragms noted on x-ray
D. polycythemia noted on complete blood cell count

A

A. FEV1/FVC ratio equal to or less than 0.70 after properly timed SABA use

How well did you know this?
1
Not at all
2
3
4
5
Perfectly
40
Q

The most effective nonpharmacologic method to prevent exacerbations in patients with COPD is:
A. weight loss for those with a BMI greater than 25 kg/m2.
B. avoid exposure to children or day-care centers.
C. brisk walking for at least 5 minutes 3-5 times a day as tolerated.
D. avoid exposure to pulmonary irritants, such as cigarette smoke.

A

D. avoid exposure to pulmonary irritants, such as cigarette smoke.

How well did you know this?
1
Not at all
2
3
4
5
Perfectly
41
Q

When managing patients with COPD who continue to smoke cigarettes, a discussion on the importance of smoking cessation should occur:
A. at the initial diagnosis visit.
B. with each COPD flare.
C. once inhaled corticosteroid therapy is initiated.
D. at every office visit.

A

D. at every office visit.

How well did you know this?
1
Not at all
2
3
4
5
Perfectly
42
Q
According to the Global Initiative for Chronic Obstructive Lung Disease (GOLD) COPD guidelines, which of the following medications is indicated for use in all COPD stages?
A. short-acting inhaled beta2-agonist
B. inhaled corticosteroid
C. long-acting anticholinergic
D. long-acting beta2 agonist
A

A. short-acting inhaled beta2-agonist

How well did you know this?
1
Not at all
2
3
4
5
Perfectly
43
Q

According to the GOLD COPD guidelines, the goal of inhaled corticosteroid use in stage III or severe COPD is to:
A. minimize the risk of repeated exacerbations.
B. improve cough function.
C. reverse alveolar hypertrophy.
D. help mobilize secretions.

A

A. minimize the risk of repeated exacerbations.

How well did you know this?
1
Not at all
2
3
4
5
Perfectly
44
Q
Which of the following systemic corticosteroid doses is most potent?
A. methylprednisolone 8 mg
B. triamcinolone 10 mg
C. prednisone 15 mg
D. hydrocortisone 18 mg
A

C. prednisone 15 mg

How well did you know this?
1
Not at all
2
3
4
5
Perfectly
45
Q
Which of the following pathogens is often implicated in a COPD exacerbation caused by respiratory tract infection?
A. Legionella species
B. Streptococcus pyogenes
C. Respiratory tract viruses
D. Staphylococcus aureus
A

C. Respiratory tract viruses

How well did you know this?
1
Not at all
2
3
4
5
Perfectly
46
Q

Which is the most appropriate choice of therapy in the treatment of a mild acute COPD exacerbation in a 42-year-old man?
A. A 5-day course of levofloxacin
B. A 7-day course of amoxicillin
C. A 10-day course of doxycycline
D. Antimicrobial therapy is usually not indicated.

A

D. Antimicrobial therapy is usually not indicated.

How well did you know this?
1
Not at all
2
3
4
5
Perfectly
47
Q

Which is the most appropriate statement about therapy for a severe COPD exacerbation in a 52-year-old man?
A. A 5-day course azithromycin should be prescribed.
B. A 10-day course of amoxicillin/clavulanate is advisable,
C. A 7-day course of trimethoprim-sulfamethoxazole is recommended.
D. The role of antimicrobial therapy is debated, even for severe disease.

A

D. The role of antimicrobial therapy is debated, even for severe disease.

How well did you know this?
1
Not at all
2
3
4
5
Perfectly
48
Q
You see a 67-year-old man with stage IV (very severe) COPD who asks, “When should I use my home oxygen?” You respond:
A. as needed when short of breath.
B. primarily during sleep hours.
C. preferably during waking hours.
D. for at least 15 hours a day.
A

D. for at least 15 hours a day.

49
Q

With a COPD exacerbation, a chest x-ray should be obtained:
A. routinely in all patients
B. when attempting to rule out a concomitant pneumonia.
C. if sputum volume is increased.
D. when work of breathing is increased.

A

B. when attempting to rule out a concomitant pneumonia.

50
Q

Which of the following best describes the role of theophylline in COPD treatment?
A. indicated in moderate to very severe COPD
B. use limited by narrow therapeutic profile and drug-drug interaction potential
C. a potent bronchodilator
D. available only in parenteral form

A

B. use limited by narrow therapeutic profile and drug-drug interaction potential

51
Q

All of the following are consistent with the GOLD COPD recommendation for pulmonary rehabilitation except:
A. reserved for very severe COPD.
B. goals include improvement in overall well being.
C. an underused therapeutic option.
D. components aimed at reducing the deconditioning common in COPD.

A

A. reserved for very severe COPD.

52
Q
Kawasaki disease most commonly occurs in what age group?
A. infants
B. children aged 2 to 3 years.
C. children approaching puberty
D. children aged 1 to 8 years
A

D. children aged 1 to 8 years

53
Q

A 6-year-old boy has a 1-year history of moderate persistent asthma that is normally well controlled with budesonide via dry powder inhaler (DPI) twice a day and the use of albuterol once or twice a week as needed for wheezing. Three days ago, he developed a sore throat, clear nasal discharge, and a dry cough. In the past 24 hours, he has had intermittent wheezing, necessitating the use of albuterol two puffs with use of an age-appropriate spacer every 3 hours with partial relief. Your next most appropriate action is to obtain:
A. a chest radiograph.
B. an oxygen saturation measurement.
C. a peak expiratory flow (PEF) measurement.
D. a sputum smear for WBCs.

A

C. a peak expiratory flow (PEF) measurement.

54
Q
You see a 4-year-old girl who has a 2-day history of signs and symptoms of an acute asthma flare resulting from viral upper respiratory tract infection. She is using inhaled budesonide and albuterol as directed and continues to have difficulty with increased occurrence of coughing and wheezing. Her respiratory rate is within 50% of upper limits of normal for her age. Her medication regimen should be adjusted to include:
A. oral theophylline.
B. inhaled salmeterol (Serevent).
C. oral prednisolone.
D. oral montelukast (Singulair).
A

C. oral prednisolone.

55
Q

Which of the following is inconsistent with the diagnosis of asthma?
A. a troublesome nocturnal cough
B. cough or wheeze after exercise
C. morning sputum production
D. colds “go to the chest” or take more than 10 days to clear

A

C. morning sputum production

56
Q

Celeste is a 9-year-old girl with moderate persistent asthma. She is not taking a prescribed inhaled corticosteroid but is using albuterol PRN to relieve her cough and wheeze. According to her mother, she currently uses about six albuterol doses per day, in particular for cough and wheeze after active play. You consider that:
A. albuterol use can continue at this level.
B. excessive albuterol use is a risk factor for asthma death.
C. she should also use salmeterol (Serevent) to reduce her albuterol use.
D. active play should be limited to avoid triggering cough and wheeze.

A

B. excessive albuterol use is a risk factor for asthma death.

57
Q
In the treatment of asthma, leukotriene modifiers should be used as:
A. long-acting bronchodilators.
B. an inflammatory inhibitor.
C. a rescue drug.
D. intervention in acute inflammation.
A

B. an inflammatory inhibitor.

58
Q
A middle-school student presents, asking for a letter stating that he should not participate in gym class because he has asthma. The most appropriate response is to:
A. write the note because gym class participation could trigger an asthma flare.
B. excuse him from outdoor activities only to avoid pollen exposure.
C. remind him that with appropriate asthma care, he should be capable of participating in gym class.
D. excuse him from indoor activities only to avoid dust mite exposure.
A

C. remind him that with appropriate asthma care, he should be capable of participating in gym class.

59
Q
After inhaled corticosteroid or leukotriene modifier therapy is initiated, clinical effects are seen:
A. immediately.
B. within the first week.
C. in about 1 to 2 weeks.
D. in about 1 to 2 months.
A

C. in about 1 to 2 weeks.

60
Q

In caring for a child with an acute asthma flare, the NP considers that, according to the National Asthma Education and Prevention Program, Expert Panel Report 3 guidelines, antibiotic use is recommended:
A. routinely.
B. with evidence of concomitant bacterial infection.
C. when asthma flares are frequent.
D. with sputum production.

A

B. with evidence of concomitant bacterial infection.

61
Q
Poorly controlled asthma in children can lead to:
A. attenuated lung development.
B. chronic tracheitis.
C. sleep apnea.
D. alveolar destruction.
A

A. attenuated lung development.

62
Q
Which of the following is not consistently performed as part of the workup for sepsis?
A. CBC with WBC differential
B. stool culture
C. blood culture
D. urine culture
A

B. stool culture

63
Q

Rates of sepsis in children have lowered in recent years mainly because of:
A. more stringent screening and diagnosis of febrile illness.
B. increased use of antipyretics.
C. longer observation period in children with febrile illness.
D. higher rates of select immunization.

A

D. higher rates of select immunization.

64
Q

The mechanism of action in fever includes which of the following?
A. an increase in systemic vascular resistance
B. endogenous pyrogens increase prostaglandin synthesis
C. immature neutrophil forms in circulation
D. atypical or reactive lymphocytes

A

B. endogenous pyrogens increase prostaglandin synthesis

65
Q

When assessing a febrile child, the NP considers that:
A. even minor temperature elevation is potentially harmful.
B. nuchal rigidity is usually not found in early childhood meningitis.
C. fever-related seizures usually occur at the peak of the temperature.
D. most children with temperatures of 38.3°C to 40°C (101°F to 104°F) have a potentially serious bacterial infection.

A

B. nuchal rigidity is usually not found in early childhood meningitis.

66
Q

Which of the following is not seen during body temperature increase found in fever?
A. lower rate of viral replication
B. toxic effect on select bacteria
C. negative effect on S. pneumoniae growth
D. increased rate of atypical pneumonia pathogen replication

A

A. lower rate of viral replication

67
Q

When providing care for a febrile patient, the NP bears in mind that all of the following are true except that:
A. the use of antipyretics is potentially associated with prolonged illness.
B. consistent use of an antipyretic provides a helpful way to shorten the course of infectious illnesses.
C. fever increases metabolic demand.
D. in a pregnant woman, increased body temperature is a potential first-trimester teratogen.

A

B. consistent use of an antipyretic provides a helpful way to shorten the course of infectious illnesses.

68
Q

Concerning the use of antipyretics in a febrile young child, which of the following statements is false?
A. A child with a serious bacterial infection usually does not have fever reduction with an antipyretic.
B. The degree of temperature reduction in response to antipyretic therapy is not predictive of presence or absence of bacteremia.
C. Compared with ibuprofen, acetaminophen has a shorter duration of antipyretic action.
D. Ibuprofen should not be used if a child is also taking a macrolide antimicrobial.

A

D. Ibuprofen should not be used if a child is also taking a macrolide antimicrobial.

69
Q

When counseling the family of an otherwise healthy 2-year-old child who just had a febrile seizure, you consider the following regarding whether the child is at risk for future febrile seizures (choose all that apply):
A. The occurrence of one febrile seizure is predictive of having another.
B. Intermittent diazepam can be used prophylactically during febrile illness to reduce risk of recurrence.
C. A milder temperature elevation in a child with a history of a febrile seizure poses significant risk for future recurrent febrile and nonfebrile seizures.
D. Consistent use of antipyretics during a febrile illness will significantly reduce the risk of a future febrile seizure.

A

A. The occurrence of one febrile seizure is predictive of having another.

B. Intermittent diazepam can be used prophylactically during febrile illness to reduce risk of recurrence.

70
Q
When evaluating a child who has bacterial meningitis, the NP expects to find cerebrospinal fluid (CSF) results of:
A. low protein.
B. predominance of lymphocytes.
C. glucose at about 30% of serum levels.
D. low opening pressure.
A

C. glucose at about 30% of serum levels.

71
Q
When evaluating a child who has aseptic or viral meningitis, the NP expects to find CSF results of:
A. low protein.
B. predominance of lymphocytes.
C. glucose at about 30% of serum levels.
D. low opening pressure.
A

B. predominance of lymphocytes.

72
Q
Sepsis is defined as the:
A. clinical manifestation of systemic infection.
B. presence of bacteria in the blood.
C. circulation of pathogens.
D. allergenic response to infection.
A

A. clinical manifestation of systemic infection.

73
Q

Gina is 2 years old and presents with a 3-day history of fever, crankiness, and congested cough. Her respiratory rate is more than 50% of the upper limits of normal for age. Tubular breath sounds are noted at the right lung base. Skin turgor is normal, and she is wearing a wet diaper. She is alert, is resisting the examination as age appropriate, and engages in eye contact. Temperature is 38.3°C (101°F). Gina’s diagnostic evaluation should include:
A. chest x-ray.
B. urine culture and sensitivity measurement.
C. lumbar puncture.
D. sputum culture.

A

A. chest x-ray.

74
Q
An early indicator of hypoperfusion is:
A. an elevation in total white blood cell count.
B. dehydration.
C. capillary refill of >2 seconds.
D. nonresponsive child.
A

C. capillary refill of >2 seconds.

75
Q

As part of the evaluation in a febrile 3-year-old boy, the following white blood cell count with differential is obtained:
WBCs = 22,100/mm3
Neutrophils = 75% (normal 40% to 70%) with toxic granulation
Bands = 15% (normal 0% to 4%)
Lymphocytes = 4% (normal 30% to 40%)
Which of the following best describes the WBC with differential results?
A. leukocytosis with neutrophilia
B. leukopenia with lymphocytosis
C. lymphopenia with neutropenia
D. leukopenia with neutropenia

These results increase the likelihood that the cause of the above-mentioned child’s infection is:
A. viral.
B. parasitic.
C. fungal.
D. bacterial.
A

A. leukocytosis with neutrophilia

Second question:
D. bacterial.

76
Q
Which of the following is the most appropriate way to relieve fever and discomfort in a child with varicella?
A. ibuprofen
B. aspirin
C. acetaminophen
D. cold bath
A

C. acetaminophen

77
Q
Potential adverse events of acetaminophen in a child with fever and mild dehydration include:
A. seizure.
B. hepatotoxicity.
C. petechial rash.
D. gastric ulcer.
A

B. hepatotoxicity.

78
Q
Sam is a 4-year-old boy who presents with a 1-week history of intermittent fever, rash, and “watery, red eyes.” Clinical presentation is of an alert child who is cooperative with examination but irritable, with a temperature of 38°C (100.4°F), pulse rate of 132 bpm, and respiratory rate of 38/min. Physical examination findings include nasal crusting; dry, erythematous, cracked lips; red, enlarged tonsils without exudate; and elevated tongue papillae. The diagnosis of Kawasaki disease is being considered. Additional findings are likely to include:
A. vesicular-form rash.
B. purulent conjunctivitis.
C. peeling hands.
D. occipital lymphadenopathy.
A

C. peeling hands.

79
Q

Laboratory findings in Kawasaki disease include all of the following except:
A. sterile pyuria.
B. elevated liver enzyme levels.
C. blood cultures positive for offending bacterial pathogen.
D. elevated erythrocyte sedimentation rate.

A

C. blood cultures positive for offending bacterial pathogen.

80
Q
Long-term consequences of Kawasaki disease include:
A. renal insufficiency.
B. coronary artery obstruction.
C. hepatic failure.
D. hypothyroidism.
A

B. coronary artery obstruction.

81
Q
The cause of Kawasaki disease is:
A. fungal.
B. viral.
C. bacterial.
D. unknown.
A

D. unknown.

82
Q
An important part of the treatment of Kawasaki disease includes the use of:
A. antibiotics.
B. antivirals.
C. immune globulin.
D. antifungals.
A

C. immune globulin.

83
Q

The following are risk factors for hypertension in children and teens (choose all that apply):
A. being obese.
B. drinking whole milk.
C. being exposed to second-hand smoke.
D. watching 2 or more hours of television per day.

A

A. being obese.

C. being exposed to second-hand smoke.

84
Q

In evaluating a 9-year-old child with a healthy BMI during a well visit, a comprehensive cardiovascular evaluation should be conducted by the following methods (choose all that apply):
A. Obtain fasting lipid profile.
B. Screen for type 2 diabetes mellitus by measuring HbA1c.
C. Assess for family history of thyroid disease.
D. Assess diet and physical activity.

A

A. Obtain fasting lipid profile.

D. Assess diet and physical activity.

85
Q
At what age is it appropriate to recommend dietary changes to parents if overweight or obesity is a concern?
A. 12 months old
B. 5 years old
C. 10 years old
D. 18 years old
A

A. 12 months old

86
Q

Risk factors for dyslipidemia in children include (choose all that apply):
A. blood pressure at the 70th to 80th percentile for age.
B. breastfeeding into the toddler years.
C. family history of lipid abnormalities.
D. family history of type 2 diabetes mellitus.

A

C. family history of lipid abnormalities.

D. family history of type 2 diabetes mellitus.

87
Q
Screening cholesterol levels in children with one or more risk factors begins at what age?
A. birth
B. 2 years
C. 5 years
D. 10 years
A

B. 2 years

88
Q

An acceptable level of total cholesterol (mg/dL) in children and teens is:
A. <170 mg/dL or 9.4 mmol/L.
B. <130 mg/dL or 7.2 mmol/L.
C. 110–130 mg/dL or 6.2 mmol/L–7.2 mmol/L.
D. 130–199 mg/dL or 7.2 mmol/L–11 mmol/L.

A

A. <170 mg/dL or 9.4 mmol/L.

89
Q

You examine a 38-year-old woman who has presented for an initial examination and Papanicolaou test. She has no complaint. Her blood pressure (BP) is 154/98 mm Hg bilaterally and her body mass index (BMI) is 31 kg/m2. The rest of her physical examination is unremarkable. Your next best action is to:
A. initiate antihypertensive therapy.
B. arrange for at least two additional BP measurements during the next 2 weeks.
C. order blood urea nitrogen, creatinine, and potassium ion measurements and urinalysis.
D. advise her to reduce her sodium intake.

A

B. arrange for at least two additional BP measurements during the next 2 weeks.

90
Q

You see a 68-year-old woman as a patient who is transferring care into your practice. She has a 10-year history of hypertension, diabetes mellitus, and hyperlipidemia. Current medications include hydrochlorothiazide, glipizide, metformin, simvastatin, and daily low-dose aspirin. Today’s BP reading is 158/92 mm Hg, and the rest of her history and examination is unremarkable. Documentation from her former healthcare provider indicates that her BP has been in the range for the past 12 months. Your next best action is to:
A. prescribe an angiotensin converting enzyme inhibitor (ACEI).
B. have her return for a BP check in 1 week.
C. advise that her current therapy is adequate.
D. add therapy with an aldosterone antagonist.

A

A. prescribe an angiotensin converting enzyme inhibitor (ACEI).

91
Q
You examine a 78-year-old woman with long-standing, poorly controlled hypertension. When evaluating her for hypertensive target organ damage, you look for evidence of:
A. lipid abnormalities.
B. insulin resistance.
C. left ventricular hypertrophy.
D. clotting disorders.
A

C. left ventricular hypertrophy.

92
Q
Diagnostic testing for a patient with newly diagnosed primary hypertension diagnosis should include all of the following except:
A. hematocrit.
B. uric acid.
C. creatinine.
D. potassium.
A

B. uric acid.

93
Q

In the person with hypertension, which of the following would likely yield the greatest potential reduction in BP in a patient with a BMI of 30 kg/m2?
A. 10-kg (22-lb) weight loss
B. dietary sodium restriction to 2.4 g (6 g NaCl) per day
C. regular aerobic physical activity, such as 30 minutes of brisk walking most days of the week
D. moderation of alcohol consumption

A

A. 10-kg (22-lb) weight loss

94
Q
You see a 38-year-old African-American male with hypertension who is currently being treated with thiazidetype diuretic. His current blood pressure reading is 156/94 mm Hg and he has no history of diabetes mellitus or chronic kidney disease. Following current best evidence, you consider adding which of the following medications?
A. ACE inhibitor
B. angiotensin receptor blocker
C. beta-adrenergic receptor antagonist
D. calcium channel blocker
A

D. calcium channel blocker

95
Q
Nondihydropyridine calcium channel blockers are contraindicated in patients with:
A. type 1 diabetes mellitus.
B. history of venous thromboembolism.
C. severe left ventricular dysfunction.
D. concomitant treatment with an ACEI.
A

C. severe left ventricular dysfunction.

96
Q

In obtaining an office BP measurement, which of the following is most reflective of the best practice?
A. Patient should sit in chair with feet flat on floor for at least 5 minutes before obtaining the reading.
B. The BP cuff should not cover more than 50% of the upper arm.
C. The patient should sit on the edge of the examination table without arm support to enhance reading accuracy.
D. Obtaining the BP reading immediately after the patient walks into the examination room is recommended.

A

A. Patient should sit in chair with feet flat on floor for at least 5 minutes before obtaining the reading.

97
Q
A BP elevation noted only at an office visit is commonly known as \_\_\_\_\_\_\_\_\_\_\_\_\_ hypertension.
A. provider-induced
B. clinical
C. white coat
D. pseudo
A

C. white coat

98
Q

The most important long term goal of treating hypertension is to:
A. strive to reach recommended numeric BP measurement.
B. avoid disease-related target organ damage.
C. develop a plan of care with minimal adverse effects.
D. treat concomitant health problems often noted in the person with this condition.

A

B. avoid disease-related target organ damage.

99
Q
You start a patient with hypertension who is already receiving an ACEI on spironolactone. You advise the patient to return in 4 weeks to check which of the following laboratory parameters?
A. sodium
B. calcium
C. potassium
D. chloride
A

C. potassium

100
Q

A 68-year-old woman presents with hypertension and BP of 152–158/92–96 mm Hg documented over 2 months on three different occasions. Electrocardiogram (ECG) and creatinine are normal, and she has no proteinuria. Clinical findings include the following: BMI 26.4 kg/m2; no S3, S4, or murmur; and point of maximal impulse at fifth intercostal space, mid-clavicular line. Which of the following represents the best intervention?
A. Initiate therapy with metoprolol.
B. Initiate therapy with hydrochlorothiazide.
C. Initiate therapy with methyldopa.
D. Continue to monitor BP, and start drug therapy if evidence of target organ damage.

A

B. Initiate therapy with hydrochlorothiazide.

101
Q

Which of the following can have a favorable effect on a comorbid condition in a person with hypertension?
A. chlorthalidone in gout
B. propranolol with airway disease
C. aldosterone antagonist in heart failure
D. methyldopa in an older adult

A

C. aldosterone antagonist in heart failure

102
Q
According to JNC-8 guidelines, all of the following medications are first-line agents for use in a middle-aged white man without diabetes mellitus except:
A. lisinopril.
B. hydrochlorothiazide.
C. metoprolol.
D. amlodipine.
A

C. metoprolol.

103
Q

You see a 59-year-old man with poorly controlled hypertension. On physical examination, you note grade 1 hypertensive retinopathy. You anticipate all of the following will be present except:
A. patient report of acute visual change.
B. narrowing of the terminal arterioles.
C. sharp optic disc borders.
D. absence of retinal hemorrhage.

A

A. patient report of acute visual change.

104
Q

According to JNC-8, a 52-year-old well woman with a healthy BMI whose blood pressure is consistently 130–135/82–86 mm Hg is considered to have:
A. normal blood pressure.
B. hypertension requiring therapy with a CCB.
C. hypertension requiring therapy with an alpha blocker.
D. hypertension requiring therapy with a thiazide-type diuretic.

A

A. normal blood pressure.

105
Q

Which of the following is associated with the highest risk of ischemic heart disease?
A. presence of microalbuminuria plus heavy alcohol intake
B. absence of microalbuminuria plus use of a thiazolidinedione
C. absence of microalbuminuria plus chronic physical inactivity
D. presence of microalbuminuria plus cigarette smoking

A

D. presence of microalbuminuria plus cigarette smoking

106
Q
What is the blood pressure goal for this patient: a 57-year-old white male with no history of diabetes mellitus (DM) or chronic kidney disease (CKD)?
A. <130/80 mm Hg
B. <140/80 mm Hg
C. <140/90 mm Hg
D. <150/90 mm Hg
A

C. <140/90 mm Hg

107
Q
What is the blood pressure goal for this patient: a 62-year-old African-American male with diabetes mellitus?
A. <130/80 mm Hg
B. <140/80 mm Hg
C. <140/90 mm Hg
D. <150/90 mm Hg
A

C. <140/90 mm Hg

108
Q
What is the blood pressure goal for this patient: a 67-year-old female with CKD?
A. <130/80 mm Hg
B. <140/80 mm Hg
C. <140/90 mm Hg
D. <150/90 mm Hg
A

C. <140/90 mm Hg

109
Q
What is the blood pressure goal for this patient: a 62-year-old female with no history of DM or CKD?
A. <130/80 mm Hg
B. <140/80 mm Hg
C. <140/90 mm Hg
D. <150/90 mm Hg
A

D. <150/90 mm Hg

110
Q
What is the blood pressure goal for this patient: an 82-year-old male with no history of DM or CKD?
A. <130/80 mm Hg
B. <140/80 mm Hg
C. <140/90 mm Hg
D. <150/90 mm Hg
A

D. <150/90 mm Hg

111
Q
What is the blood pressure goal for this patient: a 72-year-old female with DM and CKD?
A. <130/80 mm Hg
B. <140/80 mm Hg
C. <140/90 mm Hg
D. <150/90 mm Hg
A

C. <140/90 mm Hg

112
Q

You see a 62-year-old man without chronic kidney disease or diabetes mellitus who is currently being treated with low dose HCTZ and losartan. His blood pressure is currently 162/88 mm Hg. All of the following are appropriate next courses of action except:
A. increasing the dose of losartan.
B. adding a beta-adrenergic receptor antagonist.
C. adding a calcium channel blocker.
D. increasing the dose of HCTZ.

A

B. adding a beta-adrenergic receptor antagonist.

113
Q

Which of the following statements concerning postural hypotension in the elderly is false?
A. It increases the risk of falls and syncope.
B. It is characterized by a drop in blood pressure when going from a standing to a sitting position.
C. It increases the risk of cardiovascular events.
D. It is associated with the use of vasodilating medications.

A

B. It is characterized by a drop in blood pressure when going from a standing to a sitting position.

114
Q
According to American College of Cardiology Foundation/American Heart Association (ACCF/AHA) guidelines, when treating elderly patients with hypertension, which of the following medications have a compelling indication for use in the following patient conditions: heart failure? Select all that apply.
A. thiazide diuretic
B. beta blocker
C. ACEI
D. angiotensin receptor blocker (ARB)
E. aldosterone antagonist
F. calcium channel blocker
A
All of them.
A. thiazide diuretic
B. beta blocker
C. ACEI
D. angiotensin receptor blocker (ARB)
E. aldosterone antagonist
F. calcium channel blocker
115
Q
According to American College of Cardiology Foundation/American Heart Association (ACCF/AHA) guidelines, when treating elderly patients with hypertension, which of the following medications have a compelling indication for use in the following patient conditions: diabetes mellitus? Select all that apply.
A. thiazide diuretic
B. beta blocker
C. ACEI
D. angiotensin receptor blocker (ARB)
E. aldosterone antagonist
F. calcium channel blocker
A
A. thiazide diuretic
B. beta blocker
C. ACEI
D. angiotensin receptor blocker (ARB)
F. calcium channel blocker
116
Q
According to American College of Cardiology Foundation/American Heart Association (ACCF/AHA) guidelines, when treating elderly patients with hypertension, which of the following medications have a compelling indication for use in the following patient conditions: angina pectoris B? Select all that apply.
A. thiazide diuretic
B. beta blocker
C. ACEI
D. angiotensin receptor blocker (ARB)
E. aldosterone antagonist
F. calcium channel blocker
A

B. beta blocker

F. calcium channel blocker

117
Q
According to American College of Cardiology Foundation/American Heart Association (ACCF/AHA) guidelines, when treating elderly patients with hypertension, which of the following medications have a compelling indication for use in the following patient conditions: coronary artery disease? Select all that apply.
A. thiazide diuretic
B. beta blocker
C. ACEI
D. angiotensin receptor blocker (ARB)
E. aldosterone antagonist
F. calcium channel blocker
A

A. thiazide diuretic
B. beta blocker
C. ACEI
F. calcium channel blocker

118
Q
According to American College of Cardiology Foundation/American Heart Association (ACCF/AHA) guidelines, when treating elderly patients with hypertension, which of the following medications have a compelling indication for use in the following patient conditions: aortic aneurysm? Select all that apply.
A. thiazide diuretic
B. beta blocker
C. ACEI
D. angiotensin receptor blocker (ARB)
E. aldosterone antagonist
F. calcium channel blocker
A

A. thiazide diuretic
B. beta blocker
C. ACEI
F. calcium channel blocker

119
Q
According to American College of Cardiology Foundation/American Heart Association (ACCF/AHA) guidelines, when treating elderly patients with hypertension, which of the following medications have a compelling indication for use in the following patient conditions: recurrent stroke prevention? Select all that apply.
A. thiazide diuretic
B. beta blocker
C. ACEI
D. angiotensin receptor blocker (ARB)
E. aldosterone antagonist
F. calcium channel blocker
A

A. thiazide diuretic
C. ACEI
D. angiotensin receptor blocker (ARB)
F. calcium channel blocker